Real Analysis Least Upper Bound Question

In summary, to prove that lub(x + y : x in S_1, y in S_2) = lub(S_1) + lub(S_2), you need to show that for any \epsilon > 0, there exist elements x in S_1 and y in S_2 such that x + y > lub(S_1) + lub(S_2) - \epsilon. This can be done using the least upper bound property and manipulating inequalities.
  • #1
utstatistics
8
0

Homework Statement



If S1, S2 are nonempty subsets of ℝ that are bounded from above, prove that

l.u.b. {x+y : x \in S1, y \in S2 } = l.u.b. S1 + l.u.b. S2

Homework Equations



Least Upper Bound Property

The Attempt at a Solution



Using the least upper bound property, let us suppose that a is an upper bound for S1 and b is an upper bound for S2, as they are both a set of real numbers. Then there exists an x in S1 s.t. x≤a and there exists a y in S2 s.t. y≤b. By adding them together, x+y≤a+b where a and b are the upper bounds for their respected set of real numbers. Thus, it is equal to l.u.b. S1 + l.u.b. S2

I think this is how it goes. I'm sorry if my proof is horrible; it has been a significant period of time since I've done proofs, as statistics and probability classes don't use proofs! Also, I can't get \in to work. Sorry about that!
 
Physics news on Phys.org
  • #2
utstatistics said:

Homework Statement



If S1, S2 are nonempty subsets of ℝ that are bounded from above, prove that

l.u.b. {x+y : x \in S1, y \in S2 } = l.u.b. S1 + l.u.b. S2

Homework Equations



Least Upper Bound Property

The Attempt at a Solution



Using the least upper bound property, let us suppose that a is an upper bound for S1 and b is an upper bound for S2, as they are both a set of real numbers. Then there exists an x in S1 s.t. x≤a and there exists a y in S2 s.t. y≤b. By adding them together, x+y≤a+b where a and b are the upper bounds for their respected set of real numbers. Thus, it is equal to l.u.b. S1 + l.u.b. S2

I think this is how it goes. I'm sorry if my proof is horrible; it has been a significant period of time since I've done proofs, as statistics and probability classes don't use proofs! Also, I can't get \in to work. Sorry about that!

All that you have shown thus far is that if [itex]x \in S_1[/itex] and [itex]y \in S_2[/itex], then [itex]x + y \leq lub(S_1) + lub(S_2)[/itex].

This implies that [itex]lub(x + y : x\in S_1, y\in S_2) \leq lub(S_1) + lub(S_2)[/itex].

However, you still have to prove the opposite inequality.
 
  • #3
Wait, sorry if I don't understand. Are you saying I have to prove the inequality when instead of it being ≤, I prove ≥?
 
  • #4
utstatistics said:
Wait, sorry if I don't understand. Are you saying I have to prove the inequality when instead of it being ≤, I prove ≥?

Yes. If your goal is to prove that A = B, and you have only proved A <= B, then you also need to show that A >= B before you can conclude that A = B.
 
  • #5
...oh. WELL, excuse me while I think about this some more and get back to you. Haha.
 
  • #6
So let me get this straight. I now have to prove that x ≥ a in S1 and y ≥ b in S2. BUT, isn't that a contradiction? If you add them, you get x+y≥a+b, but if the elements of x, y in their respective sets are greater than the l.u.b., then a and b are not the l.u.b.

I don't quite understand. Perhaps you can guide me in the right direction. Much thanks.

-J
 
  • #7
utstatistics said:
So let me get this straight. I now have to prove that x ≥ a in S1 and y ≥ b in S2. BUT, isn't that a contradiction? If you add them, you get x+y≥a+b, but if the elements of x, y in their respective sets are greater than the l.u.b., then a and b are not the l.u.b.

I don't quite understand. Perhaps you can guide me in the right direction. Much thanks.

-J

No, you have to show that [itex]lub(x + y : x \in S_1, y \in S_2) \geq lub(S_1) + lub(S_2)[/itex]

I'm going to introduce some shorthand so it's easier to type:

a = lub(S_1)
b = lub(S_2)
c = lub(x+y : x in S_1, y in S_2)

If there were specific elements x in S_1 and y in S_2 such that x + y >= a + b, then you'd be done, because x + y <= c. But there might not be such elements. In fact, if there were, then you would have x = a and y = b. Thus each of the two sets S_1 and S_2 would contain its supremum. This is not guaranteed.

However, if you let [itex]\epsilon > 0[/itex], can you find x in S_1 and y in S_2 such that [itex]x > a - \epsilon[/itex] and [itex]y > b - \epsilon[/itex]? i.e. you might not be able to find elements that achieve the upper bound, but can you get them arbitrarily close? If so, what can you do with that?
 
Last edited:

What is the definition of a least upper bound?

A least upper bound, also known as a supremum, is the smallest number that is greater than or equal to all the elements in a set. In other words, it is the smallest upper bound for a set of numbers.

How is the least upper bound used in real analysis?

In real analysis, the least upper bound plays a crucial role in proving the existence of a limit, continuity, and differentiability of functions. It also helps in defining the completeness of a set of numbers and the convergence of sequences and series.

Does every set of real numbers have a least upper bound?

No, not every set of real numbers has a least upper bound. For example, the set of all negative real numbers does not have a least upper bound. A set must be bounded above to have a least upper bound.

What is the difference between a least upper bound and a maximum?

A least upper bound is the smallest number that is greater than or equal to all elements in a set, while a maximum is the largest number in a set. A set may have a maximum but not a least upper bound, or it may have both a maximum and a least upper bound.

How do you find the least upper bound of a set of numbers?

To find the least upper bound of a set of numbers, you can use a few different methods such as the completeness axiom, the monotone convergence theorem, or the Bolzano-Weierstrass theorem. These methods involve examining the properties of the set and its elements to determine the smallest upper bound.

Similar threads

  • Calculus and Beyond Homework Help
Replies
1
Views
1K
  • Calculus and Beyond Homework Help
Replies
1
Views
893
  • Calculus and Beyond Homework Help
Replies
1
Views
1K
  • Calculus and Beyond Homework Help
Replies
33
Views
2K
  • Calculus and Beyond Homework Help
Replies
2
Views
2K
  • Calculus and Beyond Homework Help
Replies
4
Views
1K
  • Calculus and Beyond Homework Help
Replies
11
Views
3K
  • Calculus and Beyond Homework Help
Replies
25
Views
2K
  • Advanced Physics Homework Help
Replies
2
Views
819
  • Calculus and Beyond Homework Help
Replies
17
Views
1K
Back
Top